LSAT and Law School Admissions Forum

Get expert LSAT preparation and law school admissions advice from PowerScore Test Preparation.

 Administrator
PowerScore Staff
  • PowerScore Staff
  • Posts: 8916
  • Joined: Feb 02, 2011
|
#35092
Complete Question Explanation
(See the complete passage discussion here: lsat/viewtopic.php?t=14182)

The correct answer choice is (A)

As discussed in relation to earlier questions associated with this passage set, Passage A set out a two-principle theory of justice regarding property, while Passage B applied the principle of justice in transfer to the case of Native American land ownership. The correct answer choice will describe this distinction.

Answer choice (A): This is the correct answer choice. The general view espoused by Passage A was the theory of justice regarding property, including the principle of justice in transfer. What makes this answer choice a bit difficult is the fact that although Passage B appears to be applying the principle of justice in transfer, it is not clear that the author would agree with the position taken by Passage B. Passage B talks about the "natural" way of reasoning in the abstract - "one" might take that position, but that doesn't mean the author would. It's possible that he sees another, less obvious way of reasoning and agrees with that one instead.

Answer choice (B): Passage A does not discuss competing views, but rather details only one view, the theory of justice in property regarding property.

Answer choice (C): We cannot say that the theory of justice regarding property is a commonly held principle, and Passage A does not argue in favor of a particular policy recommendation. So, this answer choice is incorrect.

Answer choice (D): Here, the answer choice is incorrect because Passage A does not provide an argument in favor of a view. Rather, Passage A simply details a view, that of the theory of justice regarding property. Further, it is incorrect to say that Passage B does not provide an argument in support of a view, since it provides reasons for the position that land in North America should be returned to Native Americans where feasible to do so.

Answer choice (E): It is incorrect to say that Passage B attempts to undermine the theory of justice regarding property detailed in Passage A.
 Cking14
  • Posts: 35
  • Joined: Mar 30, 2015
|
#19989
Hi,

I chose answer choice (D) for this one, and the correct answer is (A). I did not see any argument in passage B regarding the statement that was made - nothing about whether or not is was "endorsed." Also, Passage A does states a view, but I thought it provided a lot of details. (A) was the first to go when I did this question. What am I missing?

Please help! Thanks!
Chris
 David Boyle
PowerScore Staff
  • PowerScore Staff
  • Posts: 836
  • Joined: Jun 07, 2013
|
#20008
Cking14 wrote:Hi,

I chose answer choice (D) for this one, and the correct answer is (A). I did not see any argument in passage B regarding the statement that was made - nothing about whether or not is was "endorsed." Also, Passage A does states a view, but I thought it provided a lot of details. (A) was the first to go when I did this question. What am I missing?

Please help! Thanks!
Chris
Hello,

There are not too many details in passage A; it's mostly theory. In passage B, whether there's endorsement or not, there is some argument, e.g., " Ideally, the land should be restored to its rightful owners. This may be impractical; compromises might have to be made. But the original wrong can most easily be righted by returning the land to them—or by returning it wherever that is feasible.". So answer A is the best.

David
 NeverMissing
  • Posts: 35
  • Joined: Feb 21, 2017
|
#34624
I ruled out the correct answer—answer choice A— because it refers to passage B sketching an argument it does not necessarily endorse.

I ruled out this characterization of passage B largely because it seemed that the passage included moments of editorializing by the author that I interpreted to mean the author was, in fact, endorsing the argument (s)he put forth. In line 48, the author claims that the reasoning (s)he subsequently puts forth in the following paragraph to explain the Native American claims to lost land is "one natural (one might say obvious) way of reasoning." Later, in lines 58-59, the author claims that "the original wrong can be most easily righted by returning the land to [the Native Americans]."

Taken together, I felt these show that there is little doubt about whether or not the author endorses the argument—(s)he most certainly does! The author sees the reasoning as natural and obvious. (S)he sees the past taking of Native American lands as an "original wrong." I do not see how concessions about the argument's practicality nullify this endorsement; they merely qualify the endorsement.

Can you please explain how this passage leaves doubt as to whether or not the author endorses the argument? I've read it several times but am having trouble seeing the tone of the author as anything less than an endorsement of the argument put forth in the passage.
 Luke Haqq
PowerScore Staff
  • PowerScore Staff
  • Posts: 747
  • Joined: Apr 26, 2012
|
#34670
Hi NeverMissing!

I can definitely understand some confusion on this one--I had to skim Passage B a couple times before it became more apparent why (A) makes sense.

I think it's clear--as in the examples you referenced--that Passage B has a strong tone. To me, the critical part is around line 50:
  • Several suits have been (45) initiated by Native American tribes for recovery of lands held by them when the Nonintercourse Act took effect. One natural (one might almost say obvious) way of reasoning about Native American claims to land in (50) North America is this: Native Americans were the first human occupants of this land. Before the European invasion of North America, the land belonged to them.
The bolded part seems key because it shows that the author of Passage B is introducing a viewpoint. After the colon, there's definitely a strong view being presented, but we don't know if it's the author's view or not. This is why (A) is right--"passage B sketches an argument that it does not necessarily endorse." I understand that the fact that the author says "one might almost say obvious" seems to indicate the author's viewpoint (i.e., in agreement with what comes after the colon). But try looking at it this way: that language is a way for the author to insinuate--without explicitly stating--agreement, and the fact that it's only insinuated is why (A) is correct.

Thus, in the end you're right--it makes sense to read it and not be able to see "anything less than an endorsement," as you write. But it's the fact that it's insinuated, implied but not stated that makes (A) true. The author of B "does not necessarily endorse" what comes after the colon, even though the author's tone may suggest or insinuate endorsement.

Hope that helps!
 NeverMissing
  • Posts: 35
  • Joined: Feb 21, 2017
|
#34794
Thanks, Luke! I still don't think the phrasing of the answer choice is particularly clear, but I can see how insinuation may be at play here rather than a direct endorsement. I think this is one of those questions where you are more likely to find the correct answer by eliminating the four worse answers than by identifying the correct answer and feeling confident about its correctness.
 snowy
  • Posts: 73
  • Joined: Mar 23, 2019
|
#63737
Hi!

I'm confused as to which author/passage the "it" in answer choice A is referring to. In the explanation at the very beginning of this thread by Administrator, the explanation for why A is correct sees the "it" as referring to Author A by talking about why it is true that author A doesn't necessarily endorse the argument of Passage B. Contrarily, the explanation of Luke above has "it" referring to Author B. Can someone clarify this please, and also perhaps explain how we can tell what an unclear reference like that means in the future? Thank you in advance!

Also, in regards to the answer choice given for why answer choice C is incorrect: isn't the last paragraph of passage A a policy recommendation of sorts? And is there anything to suggest that the principles discussed aren't commonly held? The tone of the passage and the foundation from a logical justice theory made it feel like the principles were commonly held to me, which is why I chose C as my answer. In regards to the established authorities for Passage B, I saw that as the US Congress.
 Adam Tyson
PowerScore Staff
  • PowerScore Staff
  • Posts: 5153
  • Joined: Apr 14, 2011
|
#63847
The "it" in answer A most definitely refers to Passage B, snowy, and I see where our explanation muddies the water a bit on that and will take steps to clarify it. Thanks for pointing that out.

Passage A makes no policy recommendation, but only talks about what "would" happen. There is no next step taken of "so, here's what I suggest we do about it."

Other problems with answer C are that passage A tells us nothing about whether the principles being discussed are "commonly held", and passage B does not rely on any "established authorities" to support its claims (and it's not clear what those claims are - author B isn't making much pf an argument here, but just describing an argument). If we were to select answer C, we would have to be able to identify those authorities. Who says that the natural, obvious view is that the Native Americans were the first human occupants of the land? What authority claims that the current owners lack a well-founded right to it? Answer C fails on several counts.
 snowy
  • Posts: 73
  • Joined: Mar 23, 2019
|
#63874
Thank you, Adam! The "it" reference in the explanations above are definitely clearer now, as is the issue of the policy recommendation (or lack thereof, rather).

Your explanation about the policy recommendation issue clarified why C is wrong. I appreciate your help! I did want to follow up on the "established authorities" and "commonly held" aspects though, for general purposes. Does a passage need to specifically says ideas are commonly held in order for us to assume they are? In this case, the uses of the term principles and the tone of the author made me think that they were. Similarly, fore "established authorities," would Congress not fulfill that description? Thank you again, Adam!
 ShannonOh22
  • Posts: 70
  • Joined: Aug 15, 2019
|
#71066
Well this question is frustrating, to put it gently. A was immediately out because of the "sketches an argument that it does not necessarily endorse"...as mentioned in previous comments on this thread, there is a very clear tone in passage B that implies the author is in agreement with what he is "arguing". I chose D, partially because A was so outlandish in its claim that the author didn't "necessarily endorse" his/her argument, so I thought I must have missed something, and perhaps there was no argument even being made in passage B.

D...passage A "briefly states a view and then provides an argument for it"...to me, this seemed an accurate description of what happens in A - starts off with "there are two principles fundamental to a theory of justice regarding property"...briefly defines the two principles, and then follows it with an argument about how the two principles are likely not adequate to cover real-life situations, especially those in which injustice has existed. The last paragraph of A even poses a question, and then concludes "actual ownership of property must then be brought into conformity with this description".
passage B provides a detailed statement of a view around the transfer of Native American lands, and if A is any indication of how we are to classify the passage, I don't see it as having an argument at all, especially if we are to assume the author doesn't "endorse" with what he/she is saying?

My personal frustration aside, I guess I'm looking for a little bit of direction on how to best handle a question like this. I can get behind the first part of Answer Choice A - "Passage A espouses a general view without providing details" more strongly than I would agree with any of the other descriptions in any of the other answer choices about either of the passages. Should this be enough for me to just select A and move along?

This was one of 2 questions I missed in the entire RC section (the other Q being 23, in which I also chose D because both passages seemed to clearly discuss "practicability of rectification of past injustice", but that's a fight for another day).

Any insight or advice on a better way to approach a Q like this in RC is greatly appreciated!!!

Get the most out of your LSAT Prep Plus subscription.

Analyze and track your performance with our Testing and Analytics Package.